Jada swims 4 laps, which is 2/5
of the number of laps she plans to swim.

How many laps does Jada plan to swim? Use x
for the number of laps Jada plans to swim.

Equation:
Solution:

Answers

Answer 1

Let x be the number of laps Jada plans to swim.

According to the problem, 4 laps is 2/5 of the number of laps she plans to swim, so we can write:

4 = 2/5 * x

To solve for x, we can first multiply both sides by the reciprocal of 2/5, which is 5/2:

4 * 5/2 = x

Simplifying the left side:

10 = x

Therefore, Jada plans to swim 10 laps.


Related Questions

An investment of $600 is made into an account that earns 6. 5% annual simple interest for 15

years. Assuming no other deposits or withdrawals are made, what will be the balance in the

account?

Answers

According to the investment, after 15 years, the balance in the account would be $1185.

To calculate the final balance after 15 years, we can use the formula for simple interest:

Simple Interest = Principal x Interest Rate x Time

In this case, the principal is $600, the interest rate is 6.5%, and the time is 15 years.

Simple Interest = $600 x 0.065 x 15

Simple Interest = $585

So the investment of $600 earns $585 in simple interest over 15 years. To find the final balance, we add the interest earned to the initial investment:

Final Balance = Principal + Simple Interest

Final Balance = $600 + $585

Final Balance = $1185

To know more about investment here

https://brainly.com/question/365124

#SPJ4

How do you solve this equation in Derive Linear Equations - Quiz - Level H: What is the slope of the line?

Answers

The slope of the given line is 1/3 for the line passes through (0, 2) and (3, 3).

The slope of a line represents the rate at which it rises or falls as it moves horizontally from left to right.

It is calculated by taking the difference in the y-coordinates (vertical change) and dividing it by the difference in the x-coordinates (horizontal change) of two points on the line.

The slope can also be thought of as the tangent of the angle formed by the line and the positive x-axis.

In this problem, we are given that the slope of the line is 1/3.

We can use this information to find the slope between any two points on the line.

Let's consider the points (0, 2) and (3, 3) that lie on the line.

The change in y-coordinates is 3 - 2 = 1, and the change in x-coordinates is 3 - 0 = 3.

Therefore, the slope between these two points is 1/3.

Learn more about the slope at

https://brainly.com/question/3605446

#SPJ4

The question is -

What is the slope of the line?

Derive Linear Equations-Quiz-Level H

Decide if the following situation is a permutation or combination and solve. A coach needs five starters from the team of 12 players. How many different choices are there?

Answers

Answer: This situation involves choosing a group of 5 players out of a total of 12 players, where the order in which the players are chosen does not matter. Therefore, this is an example of a combination problem.

The number of ways to choose a group of 5 players out of 12 is given by the formula for combinations:

n C r = n! / (r! * (n-r)!)

where n is the total number of players, r is the number of players being chosen, and "!" represents the factorial operation.

In this case, we have n = 12 and r = 5, so the number of different choices of starters is:

12 C 5 = 12! / (5! * (12-5)!)

= 792

Therefore, there are 792 different choices of starters that the coach can make from the team of 12 players.

Step-by-step explanation:

pls hurry

2. INTEREST Marty invested $2000 in an account that pays at least 4% annual interest. He wants to see how much money he will have over the next few years. Graph the inequality y ≥ 2000 (1 + 0.04) ^× to show his potential earnings.

Answers

For the given inequality substitute the coordinates of x and y to form the graph of  y ≥ 2000 (1 + 0.04)ˣ.

What is compound interest?

Interest that is calculated on both the initial principle and any accrued interest is known as compound interest. This implies that the interest for the subsequent period is computed using the new total amount after the interest for the prior month has been added to the principal. If the interest rate and the duration are both high, the interest might add up to a sizable amount over time. Savings accounts, investments, loans, and other financial instruments frequently use compound interest.

For the given inequality we have:

For x = 0, y = 2000.

For x = 1, y = 2000(1 + 0.04) = 2080.

Plot the points on the graph to determine the graph.

Learn more about compound interest here:

https://brainly.com/question/29335425

#SPJ1

Write the number in numerical form:
Six hundred six and four thousand, three hundred fifteen ten-thousandths

Answers

The number "Six hundred six and four thousand, three hundred fifteen ten-thousandths" is equal to 606.4315 in numerical form.

Writing the number in numerical form:

Given that

Six hundred six and four thousand, three hundred fifteen ten-thousandths

To write this number in numerical form, we need to add the values of each of its digits:

6 × 100 + 0 × 10 + 6 × 1 + 4 × 0.1 + 3 × 0.01 + 1 × 0.005 = 606.4315

Therefore, the number "Six hundred six and four thousand, three hundred fifteen ten-thousandths" is equal to 606.4315 in numerical form.

Read more about place value at

https://brainly.com/question/25137147

#SPJ1

Process _____ is based on comparing the current amount of outcome variation against outcome specifications.

Answers

SPC is widely used in manufacturing, healthcare, and other industries to improve quality and efficiency.

The process you are referring to is called Statistical Process Control (SPC), which involves monitoring and analyzing a process to ensure it is within the acceptable range of variation. This is done by comparing the current amount of outcome variation against the established outcome specifications, allowing for corrective action to be taken if necessary. SPC is widely used in manufacturing, healthcare, and other industries to improve quality and efficiency.

learn more about Statistical Process Control (SPC)

https://brainly.com/question/14703158

#SPJ11

Process control is based on comparing the current amount of outcome variation against outcome specifications.

Process control is a management approach that involves monitoring and adjusting a process to ensure that it meets its intended outcome specifications.

The goal of process control is to maintain consistent outcomes over time, despite natural variation in the inputs or other factors that may affect the process.

In order to achieve this goal, process control typically involves monitoring the process using statistical methods and comparing the current level of variation in the process outcomes against the desired outcome specifications.

If the variation is within acceptable limits, the process is considered to be under control.

If the variation is outside of acceptable limits, the process may require adjustment or other corrective action to bring it back into control.

Overall, process control is an important tool in quality management, manufacturing, and other fields where consistent outcomes are critical to success.

By monitoring and adjusting processes over time, organizations can ensure that their products and services meet the needs and expectations of their customers, while minimizing waste, errors, and other sources of variation.

For similar questions on variation

https://brainly.com/question/13998680

#SPJ11

An 840-foot TV transmitter is secured by guy wires attached from the top of the tower to the ground. The wires are attached to the ground 130 feet from the base of the transmitter. How long are the guy wires?​

Answers

the guy wires are 850 feet long. We can use the Pythagorean theorem to solve this problem.

what is Pythagorean theorem  ?

The Pythagorean theorem is a mathematical concept that describes the relationship between the sides of a right triangle. It states that in any right triangle, the square of the length of the hypotenuse (the side opposite the right angle) is equal to the sum of the squares of the lengths of the other two sides.

In the given question,

We can use the Pythagorean theorem to solve this problem. The Pythagorean theorem states that in a right triangle, the square of the length of the hypotenuse (the longest side) is equal to the sum of the squares of the lengths of the other two sides.

In this case, the tower, the guy wires, and the ground form a right triangle. Let's call the length of the guy wires "x". Then we can set up the following equation:

x² = 840² + (130)²

Simplifying and solving for x, we get:

x² = 705600 + 16900

x² = 722500

x = √722500

x = 850

Therefore, the guy wires are 850 feet long.

To know more about Pythagorean theorem , visit:

https://brainly.com/question/14930619

#SPJ1

Find the three trigonometric ratios . If needed, reduce fractions.

Answers

In the given triangle the 3 trigonometric ratios are:

(A) Sinθ = 3/5, (B) Cosθ = 4/5, and (Tanθ = 3/4)

What are trigonometric ratios?

The trigonometric functions in mathematics are real functions that connect the right-angled triangle's angle to the ratios of its two side lengths.

They are extensively employed in all fields of geometry-related study, including geodesy, solid mechanics, celestial mechanics, and many others.

In general, arcsine, arccosine, tangent, cotangent, secant, and cosecant functions are used to express the inverses of sine, cosine, tangent, cotangent, secant, and cosecant functions.

So, according to t the given triangle, the 3 trigonometric ratios would be:

Sinθ = B/H

Sinθ = 27/45

Sinθ = 3/5

Cosθ = P/H

Cosθ = 36/45

Cosθ = 4/5

Tanθ = B/P

Tanθ = 27/36

Tanθ = 3/4

Therefore, in the given triangle the 3 trigonometric ratios are:

(A) Sinθ = 3/5, (B) Cosθ = 4/5, and (Tanθ = 3/4)

Know more about trigonometric ratios here:

https://brainly.com/question/24349828

#SPJ1

Jude is making cement for a driveway. The instructions show the amount of each ingredient to make 1 batch of cement. Complete each statement to adjust the ingredients for each new situation if Jude uses these instructions. 3 quarts of water. 4 packages of pre-mixed mortar.

Answers

To make 2 batches of cement, Jude will need 6 quarts of water and 8 packages of pre-mixed mortar.

To make 5 batches of cement, Jude will need 15 quarts of water and 20 packages of pre-mixed mortar.

What is proportion?

Ratio and fractions are the main bases on which proportion is explained. Two ratios are equal when they are expressed as a fraction in the form of a/b, ratio a:b, and then a proportion. In this case, a and b can be any two integers. The ratio and proportion are important building blocks for understanding the numerous ideas in science and mathematics.

To make 2 batches of cement, Jude will need 6 quarts of water and 8 packages of pre-mixed mortar.

To make 5 batches of cement, Jude will need 15 quarts of water and 20 packages of pre-mixed mortar.

Learn more about proportion on:

https://brainly.com/question/870035

#SPJ1

A frog catches insects for their lunch. The frog likes to eat flies and mosquitoes in a certain ratio, which the diagram shows.
A tape diagram with 2 tapes of unequal lengths. The first tape has 3 equal parts. A curved bracket above the first tape is labeled Flies. The second tape has 7 equal parts of the same size as in the first tape. A curved bracket below the second tape is labeled Mosquitoes.
A tape diagram with 2 tapes of unequal lengths. The first tape has 3 equal parts. A curved bracket above the first tape is labeled Flies. The second tape has 7 equal parts of the same size as in the first tape. A curved bracket below the second tape is labeled Mosquitoes.
The table shows the number of flies and the number of mosquitoes that the frog eats for two lunches.
Based on the ratio, complete the missing values in the table.
Day Flies Mosquitoes
Monday
15
1515
Tuesday
14
1414

Answers

To find the missing values in the table, we need to use the ratio of flies to mosquitoes, which is 3:7. This means that for every 3 flies, the frog eats 7 mosquitoes.

On Monday, the frog ate 15 insects in total. Let x be the number of flies the frog ate. Then the number of mosquitoes the frog ate would be 15 - x. We can set up the equation:

x/3 = (15 - x)/7

Solving for x, we get:

x = 45/10 = 4.5

Since the number of flies and mosquitoes must be whole numbers, we can round up the number of flies to 5 and calculate the number of mosquitoes as 15 - 5 = 10. So, the missing values in the table are:

Day | Flies | Mosquitoes
--------------------------
Monday | 5 | 10
Tuesday| 4 | 14

1. The largest of 2 integers is one more than three times the smaller. If the sum of the two integers is 37, find the larger integer.

2. For two consecutive integers, the sum of the smaller and twice the larger is 29. Find the smaller integer.

Answers

1.  we can use the first equation to find y = 3x + 1 = 3(9) + 1 = 28 So, the larger integer is 28, (2).  The smaller integer is 9. The larger integer is 10, which is the next consecutive integer.

What is an integer ?

An integer is a whole number ,neither a fraction or a decimal, in mathematics. Integers can be zero, positive, or negative.

Integer examples include -3, -2, -1, 0, 1, 2, 3, and so forth.

The letter "Z" stands for the set of integers.

In mathematics and allied disciplines such as computer science, physics, and finance, integers are utilized extensively.

What is a whole number ?

A whole number is a non-negative integer in mathematics, which indicates that it is either a positive integer or zero.

Negative and fractional numbers are not included in whole numbers. The full integers 0, 1, 2, 3, 4, and so on are examples.

In many different branches of mathematics as well as in other disciplines, such as arithmetic, algebra, and geometry, whole numbers are used. They are also employed in daily tasks like quantity calculations and object counts.

According to question :-

Let's call the smaller integer x and the larger integer y. We know that:

y = 3x + 1 (the largest of the two integers is one more than three times the smaller)

x + y = 37 (the sum of the two integers is 37)

We can use substitution to solve for y in terms of x:

x + (3x + 1) = 37

4x + 1 = 37

4x = 36

x = 9

Now we can use the first equation to find y:

y = 3x + 1 = 3(9) + 1 = 28

So the larger integer is 28.

Let's call the smaller integer x and the larger integer y. We know that:

y = x + 1 (the integers are consecutive)

x + 2y = 29 (the sum of the smaller and twice the larger is 29)

We can substitute y = x + 1 in the second equation:

x + 2(x + 1) = 29

Simplifying and solving for x:

3x + 2 = 29

3x = 27

x = 9

So the smaller integer is 9. The larger integer is 10, which is the next consecutive integer.

To learn more about integer visit:

https://brainly.com/question/15276410

#SPJ1

data collected from several entities over a period of time (minutes, hours, days, etc.) are called...

Answers

Data collected from several entities over a period of time (minutes, hours, days, etc.) are called time series data.

What is time series data?

Time series data is a type of data that is collected over a period of time, with measurements taken at regular intervals. The data is often used to track changes in a particular variable over time and to analyze patterns or trends in the data.

Examples of time series data include stock prices, temperature readings, and daily sales figures. Time series data can be used to make forecasts or predictions about future trends based on past data, and it is often analyzed using statistical methods such as trend analysis, seasonality analysis, and autocorrelation analysis.

Time series data can be represented graphically using a line chart or a scatterplot, with time on the x-axis and the variable of interest on the y-axis. Time series data can be univariate, meaning that there is only one variable being measured, or multivariate, meaning that there are multiple variables being measured at the same time.

What are statistical methods?

Statistical methods are a set of techniques and procedures used to analyze and interpret data in order to draw meaningful conclusions from it. These methods involve the collection, analysis, and interpretation of data in order to identify patterns, trends, and relationships between variables.

To know more about statistical method visit:

brainly.com/question/13184736

#SPJ1

Using trig to find a side.
Solve for x. Round to the nearest tenth, if necessary.

Answers

The side length x of the triangle to the nearest tenth is 170.3

What is the value of side length x?

The figures in the image are right-triangle.

From the diagram:

Angle θ = 20°

Opposite to angle θ = 62

Adjacent to angle θ = x

To find the value of x, we use the trigonometric ratio.

Note that: tangent = opposite / adjacent

Plug in the values

tan( 20 ) = 62 / x

Solve for x

x = 62 / tan( 20 )

x = 170.3

Therefore, the measure of side length labelled x is 170.3 units.

Learn more about trigonometric ratio here: brainly.com/question/28016662

#SPJ1

Helppp on this problem

Answers

The missing angles of the diagram are:

∠1 = 118°

∠2 = 62°

∠3 = 118°

∠4 = 30°

∠5 = 32°

∠6 = 118°

∠7 = 30°

∠8 = 118°

How to find the missing angles?

Supplementary angles are defined as two angles that sum up to 180 degrees. Thus:

∠1 + 62° = 180°

∠1 = 180 - 62

∠1 = 118°

Now, opposite angles are congruent and ∠2 is an opposite angle to 62°. Thus: ∠2 = 62°.

Similarly: ∠3 = 118° because it is congruent to ∠1

Alternate angles are congruent and ∠5 is an alternate angle to 32°. Thus:

∠5 = 32°

Sum of angle 4 and 5 is a corresponding angle to ∠2 . Thus:

∠4 + ∠5 = 62

∠4 + 32 = 62

∠4 = 30°

This is an alternate angle to ∠7 and as such ∠7 = 30°

Sum of angles on a straight line is 180 degrees and as such:

∠8 = 180 - (30 + 32)

∠8 = 118° = ∠6 because they are alternate angles

Read more about Missing Angles at: https://brainly.com/question/28293784

#SPJ1

Help please, I'm so lost

Answers

I gotchu <3

Since the vertex is (0, -3), the quadratic function can be written in vertex form as:

f(x) = a(x - 0)^2 - 3

Where 'a' is a constant that determines the shape of the parabola. Since the end behavior of the function is y --> - Infinite as x --> - infinite and y --> - Infinite as x --> + infinite, the leading coefficient 'a' must be negative.

So, f(x) = -a(x^2 - 0x) - 3

Now, using the given point (1, -7) on the parabola, we can substitute the coordinates into the function and solve for 'a'.

-7 = -a(1^2 - 0(1)) - 3

-7 = -a - 3

a = 10

Therefore, the quadratic function that satisfies the given characteristics is:

f(x) = -10x^2 - 3

Hope this helps :)

Given C(2, −8), D(−6, 4), E(0, 4), U(1, −4), V(−3, 2), and W(0, 2), and that △CDE is the preimage of △UVW, represent the transformation algebraically.

Answers

Rotate triangle △C'D'E' counterclockwise by approximately -0.785 radians about the origin:

[tex]x1' = 1 \times cos(-0.785) - (-4) \times sin(-0.785) \approx 0.436[/tex]

[tex]y1' = 1 \times sin(-0.785) + (-4) \times cos(-0.785) \approx -3.678[/tex]

[tex]x2' = -7 \times cos(-0.785) - 8[/tex]

What is the coordinate of the point?

The given point [tex]s C(2, -8), D(-6, 4),[/tex] and [tex]E(0, 4)[/tex] form the triangle △CDE, and the points U(1, -4), V(-3, 2), and W(0, 2) form the triangle △UVW, with △CDE being the preimage of △UVW.

To represent the transformation algebraically, we can use a combination of translations and rotations.

Translation:

To translate a point (x, y) by a vector (h, k), we add h to the x-coordinate and k to the y-coordinate of the point.

To transform triangle △CDE to triangle △UVW, we can first translate triangle △CDE by a vector (h, k) to obtain triangle △C'D'E', where C' = C + (h, k), D' = D + (h, k), and E' = E + (h, k).

Since the coordinates of C are (2, -8) and the coordinates of U are (1, -4), we can calculate the translation vector (h, k) as follows:

[tex]h = 1 - 2 = -1[/tex]

[tex]k = -4 - (-8) = 4[/tex]

So the translation vector is [tex](-1, 4).[/tex]

Rotation:

To rotate a point (x, y) by an angle θ counterclockwise about the origin, we use the following formulas:

[tex]x' = x \times \cos(\theta) - y times \sin(\theta)[/tex]

[tex]y' = x \times \sin(\theta) + y \times \cos(\theta)[/tex]

To transform triangle △C'D'E' to triangle △UVW, we can apply a rotation of angle θ counterclockwise about the origin to triangle △C'D'E', where C' = (x1', y1'), D' = (x2', y2'), and E' = (x3', y3'). Since the coordinates of C' are (2, -8) after translation, and the coordinates of U are (1, -4), we can calculate the rotation angle θ as follows:

[tex]\theta = atan2(y1' - y2', x1' - x2') - atan2(y1 - y2, x1 - x2)= atan2((-8 + 4) - (-4), (2 + 1) - (-6 + 3)) - atan2((-8) - (-4), 2 - (-6))[/tex]

Using a calculator, we can find θ to be approximately -0.785 radians.

So, the algebraic representation of the transformation that maps triangle [tex]\triangle CDE[/tex] to triangle [tex]\triangle UVW[/tex]  is:

Translate triangle △CDE by the vector (-1, 4) to obtain triangle △C'D'E':

[tex]C' = (2, -8) + (-1, 4) = (1, -4)[/tex]

[tex]D' = (-6, 4) + (-1, 4) = (-7, 8)[/tex]

[tex]E' = (0, 4) + (-1, 4) = (-1, 8)[/tex]

Therefore, Rotate triangle △C'D'E' counterclockwise by approximately -0.785 radians about the origin:

[tex]x1' = 1 \times cos(-0.785) - (-4) \times sin(-0.785) \approx 0.436[/tex]

[tex]y1' = 1 \times sin(-0.785) + (-4) \times cos(-0.785) \approx -3.678[/tex]

[tex]x2' = -7 \times cos(-0.785) - 8[/tex]

Learn more about coordinate here:

https://brainly.com/question/16634867

#SPJ1

Elaine's height is 75% of her father's. If her father is 180 cm tall, how tall is Elaine?

Answers

First, we need to find out 75% of Elaine's father's height:

75% of 180 cm = 0.75 x 180 cm = 135 cm

This means that Elaine's height is 135 cm, since it is 75% of her father's height.

Therefore, Elaine is 135 cm tall.

Answer:  Elaine is 135cm tall

Step-by-step explanation:

Elaine's height is 75% of her father's height.

We can calculate Elaine's height by multiplying her father's height by 75% or 0.75.

Elaine's height = 0.75 * 180 cm

                        = 135 cm

Therefore, Elaine is 135 cm tall.

Fred is constructing a 95% confidence interval to estimate the average length (in minutes) of movies he watches. His random sample of 15 movies averaged 114 minutes long with a standard deviation of 11 minutes. What critical value and standard error of the mean should he use?

Answers

Fred should use a critical value of 2.145 and a standard error of the mean of 2.84 to construct a 95% confidence interval for the average length of movies he watches

To construct a confidence interval for the population mean length of movies watched by Fred, we can use the following formula

Confidence interval = sample mean +/- (critical value) x (standard error)

where the standard error of the mean (SE) is calculated as:

SE = sample standard deviation / sqrt(sample size)

Since Fred's sample size is 15 and he wants a 95% confidence interval, we need to find the critical value for a t-distribution with 14 degrees of freedom (n-1), which can be obtained from a t-distribution table or calculator.

Using a t-distribution calculator with 14 degrees of freedom and a 95% confidence level, we find the critical value to be 2.145.

Next, we can calculate the standard error of the mean as

SE = 11 / sqrt(15) = 2.84

Learn more about confidence interval here

brainly.com/question/29680703

#SPJ4

Joshua started his own lawn care business. He donates 10% of his income to a local charity, which is recognized by the IRS as a qualified charitable organization. Joshua can claim expenses from his business and charitable donations as , and they will reduce the.

first blank having: Tax credits
or
tax reductions

second having: amount of taxable income
or
amount of tax owned

Answers

Joshua started his own lawn care business. He donates 10% of his income to a local charity, which is recognized by the IRS as a qualified charitable organization. Joshua can claim expenses from his business and charitable donations as tax reductions, and they will reduce the amount of taxable income.

The first blank should be filled with "tax reductions" and the second blank should be filled with "amount of taxable income."

The first blank should be filled with "tax reductions". Tax reductions reduce the amount of income that is subject to tax, whereas tax credits directly reduce the amount of tax owed.The second blank should be filled with "amount of taxable income." Business expenses and charitable donations are deductible from taxable income, meaning they reduce the amount of income that is subject to tax. By reducing the taxable income, the amount of tax owed will also be reduced.

Hence, the correct statement would be: "Joshua can claim expenses from his business and charitable donations as tax reductions, and they will reduce the amount of taxable income."

To know more about charitable here

https://brainly.com/question/26366198

#SPJ4

an assumption of the linear model of communication is the belief that _____.

Answers

The communication is a one-way process in which a message is conveyed from a sender to a receiver is one of the assumptions made by the linear model of communication.

The straightforward linear model of communication presents communication as a process with distinct beginnings and endings.

But this model ignores the fact that communication is frequently a more complex and involved process that needs context, feedback, and a variety of channels.

The linear model presumes that the message is clearly understood by the recipient and that it is received by the recipient free from distortion or interference.

To know more about linear model of communication, visit,

https://brainly.com/question/30419810

#SPJ4

An assumption of the linear model of communication is the belief that communication is a one-way process where a sender transmits a message to a receiver through a channel. This model assumes that communication is a linear process where the sender encodes a message, which is then sent through a channel to the receiver who decodes the message.

In the linear model, the sender encodes a message, sends it through a channel, and the receiver decodes the message. This model does not account for feedback or interaction between the sender and receiver, assuming that communication is a straightforward and direct transmission of information without any complexities or interruptions.

This model assumes that there is little to no feedback or interaction between the sender and receiver, and that the message is transmitted and received accurately without any distortion. However, this assumption is often criticized for oversimplifying the complex nature of communication and ignoring the role of context and feedback in the communication process.

To learn more about linear model of communication  : brainly.com/question/18269454

#SPJ11

under what conditions on a and b will the linear system have no solutions, one solution, infinitely many solutions?

Answers

The conditions will the linear equations have no solutions, one solution, infinitely many solutions are specified by variables and their coefficient matrix and states if the system is consistent or inconsistent.

Let us assume simple equations:

ax + by = c

ex + fy = g

Here a, b, c, e, f, and g are constants.

The different conditions are determined as:

1. No solutions: It is represented by D, If the determinant of the coefficient matrix is zero and the procedure is inconsistent, then we can assume that the system has no solutions.

2. One solution: If the coefficient matrix of any system is non-zero, then the system has one solution.

3. Infinitely many solutions: If the system is Consistent and the determinant of the coefficient matrix is zero, then the system has  Infinitely many solutions.

To learn more about Linear Equations,

https://brainly.com/question/14473457

#SPJ4

you are thinking of combining designer whey and muscle milk to obtain a 7-day supply that provides exactly 262 grams of protein and 54 grams of carbohydrates. how many servings of each supplement should you combine in order to meet your requirements?

Answers

We need approximately 12 servings of Designer Whey and 2 servings of Muscle Milk to obtain a 7-day supply that provides exactly 262 grams of protein and 54 grams of carbohydrates.

Let x be the number of servings of Designer Whey and y be the number of servings of Muscle Milk needed to obtain a 7-day supply that provides exactly 262 grams of protein and 54 grams of carbohydrates.

From the information given, we know that each serving of Designer Whey provides 20 grams of protein and 3 grams of carbohydrates, and each serving of Muscle Milk provides 16 grams of protein and 9 grams of carbohydrates.

Therefore, we can set up the following system of equations:

20x + 16y = 262

3x + 9y = 54

To solve for x and y, we can use any method of solving a system of equations. For example, we can use substitution:

From the second equation, we can solve for x in terms of y:

x = (54 - 9y)/3 = 18 - 3y

Substituting this into the first equation, we get:

20(18 - 3y) + 16y = 262

Simplifying, we get:

80y = 182

Solving for y, we get:

y = 2.275

Substituting this into the equation x = 18 - 3y, we get:

x = 12.175

To learn more about equations click on,

https://brainly.com/question/29118974

#SPJ4

an open box will be made by cutting a square from each corner of a 16-inches by 10-inches piece of cardboard and then folding up the sides. what size square should be cut from each corner in order to produce a box of maximum volume? what is that maximum volume?

Answers

The size of the square to cut is 5/3 inches and the maximum volume of the box is 266.67 cubic inches.

To find the size of the square to cut and the maximum volume, we can follow these steps:

Let's call the length of each side of the square to be cut x inches. So the dimensions of the base of the box would be (16-2x) inches by (10-2x) inches.

The height of the box would be x inches since we are folding up the sides.

The volume of the box can be found by multiplying the length, width, and height: V = (16-2x)(10-2x)x.

To find the maximum volume, we can take the derivative of V with respect to x and set it equal to zero, since the maximum volume occurs at a critical point.

After taking the derivative and simplifying it, we get the equation 24x^2 - 520x + 1600 = 0.

Solving this quadratic equation, we get x = 5/3 or x = 20/3. Since x must be less than 5 (the length of the shorter side), the only feasible solution is x = 5/3 inches.

Plugging this value of x back into the equation for the volume, we get V = (16-2(5/3))(10-2(5/3))(5/3) = 266.67 cubic inches.

Learn more about the maximum volume at

https://brainly.com/question/20866652

#SPJ4

some twins are sisters. all twins are siblings. therefore, some siblings are sisters. true or false

Answers

The statement "Some twins are sisters. All twins are siblings. Therefore, some siblings are sisters" is true.

 

Usually an illustration of a substantial deductive contention in which the conclusion takes coherently from the premises.

The primary introduction states that a few twins are sisters, which suggests that they are female twins. The moment preface states that all twins are kin, which implies that they are related by blood.

In this manner, on the off chance that a few twins are sisters and all twins are kin, it consistently takes after that a few kin are sisters.

It is imperative to note that the conclusion isn't essentially genuine for all kin, as a few kin may be brothers or mixed-gender twins. Be that as it may, the contention is still consistently substantial since the conclusion takes after coherently from the premises 

To know more about twins refer to this :

https://brainly.com/question/986349

#SPJ4

pls helpppp with this

Answers

Answer:

[tex]y = 2x - 1[/tex]

Step-by-step explanation:

We can represent this line in an equation in slope-intercept form:

[tex]y = mx + b[/tex],

where [tex]m[/tex] is the line's slope (rise over run), and [tex]b[/tex] is the y-coordinate of its y-intercept.

First, we can solve for the line's slope:

slope = rise / run = 2/1 = 2

Next, we can identify the y-coordinate of the y-intercept by looking at the vertical axis' value when the red line crosses it:

-1

Finally, we can put these two pieces of information together to form an equation in slope-intercept form:

[tex]\boxed{y = 2x - 1}[/tex]

when solving an oblique triangle given three sides, use the ---select--- form of the law of cosines to solve for an angle.

Answers

When solving an oblique triangle given three sides, use the inverse cosine form of the Law of cosines to solve for an angle.

When solving an "oblique-triangle" given three sides, we would use the inverse cosine, form of the Law of Cosines to solve for an angle.

The "Inverse-Cosine" function allows us to find the measure of an angle when given the ratio of the lengths of the triangle's sides.

The "Law-of-Cosines" relates the lengths of the sides of a triangle to the cosine of one of its angles.

The "Law-of-Cosines" states that for any triangle with sides of lengths a, b, and c, and opposite angles A, B, and C, respectively:

⇒ c² = a² + b² - 2ab × cos(C),

To solve for an angle, we would rearrange the equation to find the cosine of the angle,

⇒ Cos(C) = (a² + b² - c²)/(2ab),

Then, we will take the inverse cosine of both sides of the equation to find the value of the angle,

⇒ C = cos⁻¹((a² + b² - c²)/(2ab)),

This helps us to find the measure of angle C, depending on the units used in the original triangle sides.

Learn more about Law Of Cosines here

https://brainly.com/question/30766161

#SPJ4

The given question is incomplete, the complete question is

When solving an oblique triangle given three sides, use the _____ form of the Law of cosines to solve for an angle.

About 16% of people who get their feet examined are found to have athlete's foot. What is the probability of a podiatrist examining five people until he finds the first patient who has athlete's foot? (0. 16)^5 (0. 84)^4(0. 16)^1 (0. 16)^4(0. 84)^1 (0. 84)5 1 − (0. 16)^4

Answers

The probability of examining the five people till first patient has athlete's foot is given by (0. 84)^4(0. 16)^1.

Percent of people have athlete foot = 16%

Let us consider the probability of success represented by p = 0.16.

Using geometric distribution,

Probability of finding first patient with athlete's foot on kth examination is given by geometric probability distribution,

P(X = k) = (1 - p)^(k - 1) × p

where X is random variable representing number of trials needed until  first success.

Probability of finding the first patient with athlete's foot on the fifth examination k = 5,

⇒ P(X = 5) = (1 - p)^(5 - 1) × p

⇒ P(X = 5) = (0.84)^4 × 0.16

Simplify it we get,

⇒ P(X = 5) = 0.0473

Therefore, the probability of a podiatrist examining five people until he finds the first patient who has athlete's foot is (0. 84)^4(0. 16)^1.

learn more about probability here

brainly.com/question/15221046

#SPJ4

can someone give me the answers to these 5?? pleaseee!!

Answers

The MAD of the hourly wages given would be $ 0.48. The range would be $ 2.00. Q1 would be $8.25. Q3 would then be $9.25. The IQR would be $1.00

How to find the number summaries ?

Calculate the MAD:

First, find the mean of the data set:

mean = (sum of all values) / (number of values)

mean = (8.25 + 8.50 + 9.25 + 8.00 + 10.00 + 8.75 + 8.25 + 9.50 + 8.50 + 9.00) / 10

mean = 88.00 / 10 = 8.80

Then, find the mean of these absolute deviations:

MAD = (sum of absolute deviations) / (number of values)

MAD = (0.55 + 0.30 + 0.45 + 0.80 + 1.20 + 0.05 + 0.55 + 0.70 + 0.30 + 0.20) / 10

MAD = 4.10 / 10 = 0.41

Calculate the range:

range = maximum value - minimum value

range = 10.00 - 8.00 = 2.00

Find Q1 and Q3:

{8.00, 8.25, 8.25, 8.50, 8.50, 8.75, 9.00, 9.25, 9.50, 10.00}

Q1 is the median of the lower half, and Q3 is the median of the upper half.

Lower half: {8.00, 8.25, 8.25, 8.50, 8.50}

Upper half: {8.75, 9.00, 9.25, 9.50, 10.00}

Q1 = median of lower half = 8.25

Q3 = median of upper half = 9.25

Calculate the IQR:

IQR = Q3 - Q1

IQR = 9.25 - 8.25 = 1.00

Find out more on MAD at https://brainly.com/question/3250070

#SPJ1

in general, if sample data are such that the null hypothesis is rejected at the a 5 1% level of significance based on a two-tailed test, is h0 also rejected at the a 5 1% level of significance for a corresponding onetailed test? explain.

Answers

The directionality of the alternative hypothesis and the support offered by the sample data determine whether the null hypothesis is likewise rejected at the 5% level of significance for a related one-tailed test.

When the two-tailed test rejects the null hypothesis, it means that the sample data, regardless of how we look at it, support the null hypothesis.. A one-tailed test, however, simply considers the evidence in one way. As a result, the null hypothesis should be used if the sample data only show evidence that the alternative hypothesis is true in one direction (for example, greater than).

To know more about Null hypothesis, visit,

https://brainly.com/question/4436370

#SPJ4

Identify the expression that is not equivalent to 6x + 3.

Answers

The resultant value of the given expression x² + 10x + 24 when x = 3 is (C) 63.

What are expressions?

A finite collection of symbols that are properly created in line with context-dependent criteria is referred to as an expression, sometimes known as a mathematical expression.

Expressions in writing are made using mathematical operators such as addition, subtraction, multiplication, and division.

For instance, "4 added to 2" will have the mathematical formula 2+4.

So, we have the expression:

= x² + 10x + 24

Now, solve when x = 3 as follows:

= x² + 10x + 24

= 3² + 10(3) + 24

= 9 + 30 + 24

= 63

Therefore, the resultant value of the given expression x² + 10x + 24 when x = 3 is (C) 63.

Know more about expressions here:

https://brainly.com/question/723406

#SPJ1

Correct question:

Evaluate the expression when x = 3.

x² + 10x + 24

a. 81

b. 86

c. 63

d. 60

Other Questions
What was Martins problem that was presented to the dragon? The Booth Company's sales are forecasted to double from $1,000 in 2019 to $2,000 in 2020. Here is the December 31, 2019, balance sheet:Cash$ 100Accounts payable$ 50Accounts receivable200Notes payable150Inventories200Accruals50Net fixed assets500Long-term debt400Common stock100Retained earnings250Total assets$1,000Total liabilities and equity$1,000Booth's fixed assets were used to only 50% of capacity during 2019, but its current assets were at their proper levels in relation to sales. All assets except fixed assets must increase at the same rate as sales, and fixed assets would also have to increase at the same rate if the current excess capacity did not exist. Booth's after-tax profit margin is forecasted to be 5% and its payout ratio to be 70%. What is Booth's additional funds needed (AFN) for the coming year? Round your answer to the nearest dollar. which line in the below code causes an error? #include #include using namespace std; int main() { vector idnums(5); idnums.at(1) = 60; idnums.at(2) = 70; idnums.at(3) = 82; idnums.at(4) = 90; idnums.at(5) = 92; cout Last year Janet purchased a $1,000 face value corporate bond with an 11% annual coupon rate and a 30-year maturity. At the time of the purchase, it had an expected yield to maturity of 12.2%. If Janet sold the bond today for $1,045.79, what rate of return would she have earned for the past year? Do not round intermediate calculations. Round your answer to two decimal places. ssue advocacy is coordinated spending between an interest group and a candidate for elected office. true false The modern Keynesian Model assumes thatSince the modern Keynesian Model allows for some price response, the aggregate supply curve which of the following statements applies to the discount rate? the federal funds rate is the same as this rate. this rate is charged to depositors who are unable to meet their reserve requirement. the fed does not directly control this rate. this rate is used when banks borrow directly from the fed. CopyStateThispriorany rpermManavvttropyomandavMarven and three friends are renting a car for a trip. Rental prices areshown in the table.ItemPART BSmall car rental fee-seats 4 passengersFull-size car rental fee-seats 4 passengersInsurancePrice465=25x$39/day$49/day$21/day25(X=18.6-198018.61465LISIf they still use the coupon, how many days could they rent the small carwith insurance if they have $465 to spend? other things the same, if the fed increases the rate at which it increases the money supply then the short-run phillips curve shifts right in the long run. a. true b. false Find the three trigonometric ratios. If needed, reduce fractions. The Read Books, Share the Love advertisement effectively uses the association technique to convince people to read more books. The advertisement targets parents who could easily associate the children in the picture with their own children. This has a positive connotation of intimacy and love. The image implies that reading is not just intellectually beneficial, but that it creates a closer bond between parents and their children. Because of this association technique, parents are persuaded to read to their children. Most parents certainly would want their children to do well academically and feel loved at the same time. Therefore, this ad effectively encourages literacy.Which of the following revisions would most improve this paragraph?specific details from the postera strong topic sentencea discussion of the techniques used in the adadditional information about the ad in general Sheffield Inc. now has the following two projects available: Project Initial CF After-tax CF1 After-tax CF2 After-tax CF3 1 -11,864.01 5,250 6,125 9,500 2 -3,336.42 3,750 3,150 = - Assume that RF = 5.How do you calculate working capital for a new project? Beaver, a city in the United States, is attempting to attract a professional soccer team. Beaver is planning to build a new stadium that will cost $250 million. Annual upkeep is expected to amount to $800,000. The turf will have to be re- placed every 10 years at a cost of $950,000. Painting every 5 years will cost $75,000. If the city expects to maintain the facility indefinitely, what is the estimated capitalized cost at i = 8% per year? in a standard additions method workup what information from the linear regression is most closely related to the unknown concentration? (used to determine it) Laws passed by the federal or a state legislature and either approved or rejected by the executive branch are true or false? one of the provisions of the e-government act is that federal agencies must review their it systems for privacy risks Your broker charges $0.0029 per share per trade. The exchange charges $0.0173 per share per trade for removing liquidity and credits $0.0155 per share per trade for adding liquidity. The current best BID price for stock XYZ is $82.89 per share, while the current best ASK price is $82.90 per share. You post an order to buy XYZ at the current best BID price and wait. Shortly after, the best BID and ASK prices move lower (down) by one cent each. Your buy order is executed. Immediately, you post an order to sell XYZ at the new best BID price, and your sell order is executed. What will be your net loss per share to buy and sell XYZ after considering the commissions and any exchange fees or credits? how to solve this equetion. a scientist used 786 millileters Find the surface area and volume of the composite solid. courtney has $4.85 in nickels, quarters, and dimes. she has12 quarters and 7 nickels. how many more dimes than nickels does she have?